19.3 Lokale Extrema


19.3.1 Definition. Lokales Minimum und Maximum.
Es sei \bgroup\color{demo}$ f:E\supseteq U\to\mathbb{R}$\egroup und \bgroup\color{demo}$ \xi \in U$\egroup. Dann heißt \bgroup\color{demo}$ \xi $\egroup genau dann lokales Minimum (resp. lokales Maximum) von \bgroup\color{demo}$ f$\egroup, wenn ein \bgroup\color{demo}$ \varepsilon >0$\egroup existiert, so daß für all \bgroup\color{demo}$ x\in U$\egroup mit \bgroup\color{demo}$ \vert x-\xi \vert<\varepsilon $\egroup die Ungleichung \bgroup\color{demo}$ f(x)\geq f(\xi )$\egroup (resp. \bgroup\color{demo}$ f(x)\leq f(\xi )$\egroup) gilt. Gilt sogar die strikte Ungleichung \bgroup\color{demo}$ f(x) > f(\xi )$\egroup (resp. \bgroup\color{demo}$ f(x)<f(\xi )$\egroup) für all jene \bgroup\color{demo}$ x\ne \xi $\egroup, so heißt \bgroup\color{demo}$ \xi $\egroup ein lokales striktes Minimum (resp. lokales striktes Maximum).


19.3.6 Bemerkung. Gradient.
Da es nur Sinn macht für Skalar-wertige Funktionen \bgroup\color{demo}$ f:E\supseteq U\to\mathbb{R}$\egroup von lokalen Extrema zu sprechen wollen wir für diese eine andere Interpretation der Ableitung \bgroup\color{demo}$ f'(x)\in L(E,\mathbb{R})$\egroup geben. Wenn \bgroup\color{demo}$ E=\mathbb{R}^n$\egroup ist, dann ist die Jacobi-Matrix

\bgroup\color{demo}$\displaystyle [f'(x)] = \Bigl(\d _1 f(x),\dots,\d _n f(x)\Bigr)
$\egroup

und \bgroup\color{demo}$ f'(x)(v)$\egroup ist durch Matrix-Multiplikation gegeben:

\bgroup\color{demo}$\displaystyle f'(x)(v)= \Bigl(\d _1 f(x),\dots,\d _n f(x)\Bigr)\cdot
\begin{pmatrix}v_1 \vdots  v_n\end{pmatrix},
$\egroup

wobei \bgroup\color{demo}$ v_1,\dots,v_n$\egroup die Koordinaten von \bgroup\color{demo}$ v$\egroup sind. Wir müssen also den Vektor \bgroup\color{demo}$ v$\egroup als Spaltenvektor interpretieren und \bgroup\color{demo}$ f'(x)$\egroup als Zeilenvektor der gleichen Länge. Wenn wir die Matrix \bgroup\color{demo}$ [f'(x)]$\egroup transponieren, also \bgroup\color{demo}$ f'(x)$\egroup ebenfalls als Spalten-Vektor in \bgroup\color{demo}$ \mathbb{R}^n$\egroup auffassen, dann nennen wir diesen den Gradienten von $ f$ bei $ x$ und schreiben \bgroup\color{demo}$ \operatorname{grad}f(x)$\egroup dafür. Es gilt somit

\bgroup\color{demo}$\displaystyle f'(x)(v) =
\Bigl(\d _1 f(x),\dots,\d _n f(x)\...
..._{i=1}^n \d _i f(x)\cdot v_i
=\langle \operatorname{grad}f(x),v\rangle.
$\egroup

Die Gleichung \bgroup\color{demo}$ f'(x)(v)=\langle \operatorname{grad}f(x),v\rangle$\egroup benutzt nun keine Koordinaten (von \bgroup\color{demo}$ v$\egroup und von \bgroup\color{demo}$ \operatorname{grad}f(x)$\egroup) mehr, macht also als Definition von \bgroup\color{demo}$ \operatorname{grad}f(x)$\egroup auf Vektorräumen ohne vorgegebener Basis nur versehen mit einem skalarem Produkt Sinn.




19.3.8 Lemma. Geometrische Bedeutung des Gradientens.
Es sei \bgroup\color{demo}$ f:E\supseteq U\to\mathbb{R}$\egroup differenzierbar bei \bgroup\color{demo}$ x\in U$\egroup. Dann zeigt \bgroup\color{demo}$ \operatorname{grad}f(x)\in E$\egroup in die Richtung des größten Anstiegs von \bgroup\color{demo}$ f$\egroup.

Jede Höhenlinie von \bgroup\color{demo}$ f$\egroup, d.h. differenzierbare Kurve \bgroup\color{demo}$ c:\mathbb{R}\supseteq I\to U$\egroup mit konstanten \bgroup\color{demo}$ f\o c$\egroup, steht in jedem Punkt \bgroup\color{demo}$ c(t)$\egroup normal auf \bgroup\color{demo}$ \operatorname{grad}
f(c(t))$\egroup.

Image ..//pic-5-09.gif

Beweis. Sei \bgroup\color{demo}$ v\in E$\egroup mit \bgroup\color{demo}$ \Vert v\Vert=1$\egroup eine Richtung. Dann ist die Richtungsableitung von \bgroup\color{demo}$ f$\egroup bei \bgroup\color{demo}$ x$\egroup in Richtung \bgroup\color{demo}$ v$\egroup wegen der Ungleichung von Cauchy-Schwarz (13.6)

\bgroup\color{demo}$\displaystyle d_vf(x)=f'(x)(v)=\langle \operatorname{grad}f(...
...x)\Vert \Vert v\Vert \cos(\measuredangle(\operatorname{grad}f(x),v)).
$\egroup

Dies wird also maximal, wenn \bgroup\color{demo}$ \cos(\measuredangle(\operatorname{grad}f(x),v))=1$\egroup ist, d.h. \bgroup\color{demo}$ \measuredangle(\operatorname{grad}f(x),v)=0$\egroup ist, also \bgroup\color{demo}$ v$\egroup in die selbe Richtung wie \bgroup\color{demo}$ \operatorname{grad}f(x)$\egroup zeigt.

Der zweite Teil der Aussage folgt nun so: Da \bgroup\color{demo}$ f\o c$\egroup konstant ist folgt durch Differenzieren \bgroup\color{demo}$ 0=(f\o c)'(t)=f'\Bigl(c(t)\Bigr) \Bigl(c'(t)\Bigr)=
\langle \operatorname{grad}f(c(t)),c'(t)\rangle$\egroup, also ist \bgroup\color{demo}$ c'(t)\perp \operatorname{grad}f(c(t))$\egroup.     []


Bemerkung. Gradientenfeld.
Es sei eine \bgroup\color{demo}$ C^1$\egroup Funktion \bgroup\color{demo}$ f:E\supseteq U\to\mathbb{R}$\egroup vorgegeben. Dann können wir das Vektorfeld \bgroup\color{demo}$ \operatorname{grad}f:E\supseteq U\to E$\egroup betrachten, das sogenannte Gradientenfeld von \bgroup\color{demo}$ f$\egroup.

Image ..//pic-5-10.gif

Die Lösungen der zugehörigen gewöhnlichen Differentialgleichung

\bgroup\color{demo}$\displaystyle x'(t)=\operatorname{grad}f (x(t))
$\egroup

beschreiben dann die Wege, die Punkte nehmen, die denn stärksten Anstieg folgen. Wenn man \bgroup\color{demo}$ f$\egroup als Luftdruck interpretiert, so sind beschreiben die Lösungen dieser Differentialgleichung, welche (abgesehen von der Corioliskraft) durch die Druckverteilung \bgroup\color{demo}$ f$\egroup entstehenden Winde.

animation animation




19.3.2 Lemma. Notwendige Bedingung für Extremum.
Es sei \bgroup\color{demo}$ \xi \in U$\egroup ein lokales Extremum (d.h. Maximum oder Minimum) von \bgroup\color{demo}$ f:E\supseteq U\to\mathbb{R}$\egroup und \bgroup\color{demo}$ f$\egroup sei differenzierbar bei \bgroup\color{demo}$ \xi $\egroup oder besitze zumindest die Richtungsableitungen \bgroup\color{demo}$ d_vf(\xi )$\egroup für alle \bgroup\color{demo}$ v\in E$\egroup.

Dann ist \bgroup\color{demo}$ \xi $\egroup ein kritischer Punkt von \bgroup\color{demo}$ f$\egroup, d.h. \bgroup\color{demo}$ f'(\xi )=0$\egroup.

Image ..//pic-5-02.gif

Beweis. Für \bgroup\color{demo}$ v\in E$\egroup besitzt auch \bgroup\color{demo}$ t\mapsto f(\xi +tv)$\egroup ein lokales Extremum bei \bgroup\color{demo}$ t=0$\egroup und somit z.B. im Falle eines lokalen Minimums \bgroup\color{demo}$ f(\xi +tv)\geq f(\xi )$\egroup für alle \bgroup\color{demo}$ t$\egroup und damit

\bgroup\color{demo}$\displaystyle \frac{f(\xi +tv)-f(\xi )}{t}
\left\{\begin{ar...
...l} \geq 0 &\text{für }t>0 \\
\leq 0 &\text{für }t<0 \end{array}\right.
$\egroup

also ist \bgroup\color{demo}$ f'(\xi )(v)=\d _v f(\xi )=0$\egroup.     []




19.3.3 Proposition. Hinreichende Bedingung für lokales Extremum.
Es sei \bgroup\color{demo}$ f:E\supseteq U\to\mathbb{R}$\egroup \bgroup\color{demo}$ C^2$\egroup, \bgroup\color{demo}$ f'(\xi )=0$\egroup und die symmetrische Form \bgroup\color{demo}$ f''(\xi ):E\times E\to\mathbb{R}$\egroup sei positiv definit, d.h. \bgroup\color{demo}$ f''(\xi )(v,v)>0$\egroup für alle \bgroup\color{demo}$ v\ne 0$\egroup. Dann hat \bgroup\color{demo}$ f$\egroup bei \bgroup\color{demo}$ \xi $\egroup ein striktes lokales Minimum.

Eine bilineare symmetrische Form \bgroup\color{demo}$ b$\egroup ist genau dann positiv definit, wenn \bgroup\color{demo}$ \exists\delta >0 \forall h:b(h,h)\geq \delta \Vert h\Vert^2$\egroup: Denn die Einheitssphäre \bgroup\color{demo}$ \{h:\Vert h\Vert=1\}$\egroup ist kompakt und somit existiert \bgroup\color{demo}$ \delta :=\min\{b(h,h):\Vert h\Vert=1\}>0$\egroup und für \bgroup\color{demo}$ 0\ne v\in E$\egroup gilt \bgroup\color{demo}$ v=\Vert v\Vert h$\egroup, wobei \bgroup\color{demo}$ h:=\frac1{\Vert v\Vert}v$\egroup und damit \bgroup\color{demo}$ b(v,v)=b(\Vert v\Vert h,\Vert v\Vert h)=\Vert v\Vert^2 b(h,h)\geq \Vert v\Vert^2 \delta $\egroup.

In der Mathematik 1 haben wir gezeigt, daß eine symmetrische bilinear-Form \bgroup\color{demo}$ b:\mathbb{R}^n\times \mathbb{R}^n\to\mathbb{R}$\egroup genau dann positiv definit ist, wenn es die zugehörige quadratische Matrix \bgroup\color{demo}$ (b_{i,j})_{i,j=1}^n$\egroup mit \bgroup\color{demo}$ b_{i,j}:=b(e_i,e_j)$\egroup ist, d.h. die Determinante der Hauptminore \bgroup\color{demo}$ (b_{i,j})_{i,j=1}^k$\egroup positiv ist für jedes \bgroup\color{demo}$ 1\leq k\leq n$\egroup. Äquivalent dazu ist auch, daß alle Eigenwerte positiv sind.

Eine symmetrische bilinear-Form \bgroup\color{demo}$ b$\egroup heißt negativ definit, wenn \bgroup\color{demo}$ -b$\egroup positiv definit ist, d.h. \bgroup\color{demo}$ b(v,v)<0$\egroup für alle \bgroup\color{demo}$ v\ne 0$\egroup ist. Dies ist also genau dann der Fall, wenn alle Eigenwerte negativ, bzw. die Hauptminoren alternierendes Vorzeichen haben und zwar beginnend mit \bgroup\color{demo}$ -$\egroup für die \bgroup\color{demo}$ 1\times 1$\egroup-Minore.

Beweis. O.B.d.A. sei \bgroup\color{demo}$ \xi =0$\egroup. Nach Taylor's Theorem (19.2.1) ist

\bgroup\color{demo}$\displaystyle f(x) = f(0) + f'(0)\cdot x + \int_0^1 (1-t) f''(tx)(x,x) dt
$\egroup

Die Menge der stetigen positiv definiten symmetrischen Bilinear-Formen ist offen in allen symmetrischen Bilinear-Formen, denn wenn \bgroup\color{demo}$ b_0(x,x)\geq \delta \Vert x\Vert^2$\egroup ist und \bgroup\color{demo}$ \Vert b-b_0\Vert<\delta $\egroup ist, so ist

$\displaystyle b(x,x)$ $\displaystyle = b_0(x,x) + \Bigl(b(x,x)-b_0(x,x)\Bigr)$    
  $\displaystyle \geq b_0(x,x) - \vert b(x,x)-b_0(x,x)\vert \geq \oversetbrace{>0}\to{(\delta - \Vert b-b_0\Vert)}\Vert x\Vert^2$    

Da \bgroup\color{demo}$ x\mapsto f''(x)$\egroup stetig ist und \bgroup\color{demo}$ f''(0)$\egroup positiv definit ist existiert ein \bgroup\color{demo}$ \delta >0$\egroup und ein \bgroup\color{demo}$ \varepsilon >0$\egroup, sodaß alle \bgroup\color{demo}$ \Vert x\Vert<\varepsilon $\egroup in \bgroup\color{demo}$ U$\egroup liegen und \bgroup\color{demo}$ f''(x)(h,h)\geq \frac{\delta }2 \Vert h\Vert^2$\egroup für alle \bgroup\color{demo}$ h\in E$\egroup gilt. Somit ist für diese \bgroup\color{demo}$ x$\egroup

\bgroup\color{demo}$\displaystyle \int_0^1 (1-t) f''(tx)(x,x) dt
\geq \int_0^...
...\Vert x\Vert^2\frac{\delta }2  dt
\geq \Vert x\Vert^2\frac{\delta }4,
$\egroup

daher ist \bgroup\color{demo}$ f(x) > f(0)$\egroup für alle \bgroup\color{demo}$ 0\ne \Vert x\Vert<\varepsilon $\egroup.     []


19.3.4 Beispiele von Extremalpunkten.
Die typischen quadratischen (nicht degenerierten) Formen mit 2 positiven, resp. einem positiven und einem negativen Eigenwert, resp. zwei negativen Eigenwerten sind:

\bgroup\color{demo}$\displaystyle \begin{matrix}
x^2+y^2 \qquad\qquad & \qquad x...
...it}\qquad &\text{indefinit} &\qquad\text{negativ definit}
\end{matrix}
$\egroup

Image ..//pic-4-24a.gif

Image ..//pic-4-24b.gif

  1. $ f(x,y):=x y$ hat nur $ (0,0)$ als kritischen Punkt. Dieser ist ein Sattelpunkt.

    Image ..//pic-5-03.gif

    In der Tat ist $ f'(x,y)=(y,x)=0$ $ \Leftrightarrow$ $ x=0=y$ und die Eigenwerte $ \lambda $ von

    $\displaystyle f''(0,0)=\begin{pmatrix}0 & 1  1 & 0\end{pmatrix}$

    sind durch $ 0=\det(\lambda \operatorname{id}-f''(0,0))=\lambda ^2-1$, also $ \lambda =\pm 1$ gegeben.
  2. $ f(x,y):=x^3+y^3-3xy$ hat nur $ (0,0)$ und $ (1,1)$ als kritische Punkte. Der erste ist ein Sattelpunkt (setze $ y=0$) und der zweite ein lokales striktes Minimum.

    Image ..//pic-5-04.gif

    In der Tat ist $ f'(x,y)=3(x^2-y,y^2-x)=0$ $ \Leftrightarrow$ $ (x,y)=(0,0)$ oder $ (x,y)=(1,1)$. Die Eigenwerte $ \lambda $ von

    $\displaystyle f''(x,y)=3\begin{pmatrix}2x & -1  -1 & 2y \end{pmatrix}
$

    sind für $ (x,y)=(0,0)$ ähnlich wie zuvor $ \pm 3$ und für $ (x,y)=(1,1)$ durch $ 0=\det(\lambda \operatorname{id}-f''(0,0))=(\lambda -6)^2-3^2$, also $ \lambda =6\pm 3$ gegeben.
  3. $ f(x,y):=\sin(x)+\sin(y)+\sin(x+y)$ für $ 0\leq x,y\leq \frac{\pi}2$.

    Image ..//pic-5-05.gif

    Der einzige kritische Punkt im Inneren ist $ (\frac{\pi}3,\frac{\pi}3)$, ein globales Maximum. Am Rand sind $ (0,0)$ und $ (\frac{\pi}2,\frac{\pi}2)$ lokale Minima (und der erste Punkt ein globales) und $ (\frac{\pi}2,\frac{\pi}4)$ und $ (\frac{\pi}4,\frac{\pi}2)$ lokale Maxima am Rand aber nicht von $ f$ am ganzen Quadrat.

    Hier ist $ f'(x,y)=(\cos(x)+\cos(x+y),\cos(y)+\cos(x+y))=(0,0)$ genau dann wenn $ \cos(x)=\cos(y)$, also $ x=y$, und $ 0=\cos(x)+\cos(2x)=\cos(x)+2\cos^2(x)-1$, also $ \cos(x)=-\frac14\pm\frac34$, d.h. $ x=\arccos(\frac12)=\frac{\pi}3$. In diesem Punkt ist

    $\displaystyle f''(x,y)$ $\displaystyle = \begin{pmatrix}-\sin(x)-\sin(x+y) & -\sin(x+y)  -\sin(x+y) & -\sin(x)-\sin(x+y) \end{pmatrix}$    
      $\displaystyle = \begin{pmatrix}-\sqrt{3} & -\frac{\sqrt{3}}2  -\frac{\sqrt{3}}2 & -\sqrt{3} \end{pmatrix}$    

    mit Eigenwerten $ -\frac{3\sqrt{3}}2$ und $ -\frac{\sqrt{3}}2$, also liegt ein Maximum vor.


19.3.9 Bemerkung. Extremalpunkte unter Nebenbedingung.
Sehr oft wird die Menge \bgroup\color{demo}$ X$\egroup in der wir eine Extremalstelle von \bgroup\color{demo}$ f$\egroup suchen nicht unbedingt eine offene Teilmenge eines Vektorraums \bgroup\color{demo}$ E$\egroup sein. Falls wir \bgroup\color{demo}$ X$\egroup lokal durch eine Abbildung \bgroup\color{demo}$ h:H\to X$\egroup parametrisieren können, d.h. zu vorgegebenen \bgroup\color{demo}$ x_0\in X$\egroup eine offenen Menge \bgroup\color{demo}$ V\subseteq E$\egroup mit \bgroup\color{demo}$ x_0\in V$\egroup und offene Menge \bgroup\color{demo}$ U$\egroup eines Vektorraums \bgroup\color{demo}$ H$\egroup mit \bgroup\color{demo}$ 0\in U$\egroup sowie eine surjektive Abbildung \bgroup\color{demo}$ h:U\to V\cap X$\egroup finden können, dann ist \bgroup\color{demo}$ x_0$\egroup genau dann ein lokales Extremum von \bgroup\color{demo}$ f\vert _X:X\to \mathbb{R}$\egroup in \bgroup\color{demo}$ X$\egroup, wenn \bgroup\color{demo}$ h_0\in h^{-1}(x_0)$\egroup ein solches von \bgroup\color{demo}$ f\o h:H\supseteq U\to V\cap X\to \mathbb{R}$\egroup ist. Somit ist in dieser Situation das Problem wieder auf Funktionen zurückgespielt, die auf offenen Teilmengen von Vektorräumen definiert sind.

Bisweilen werden wir aber eine Parametrisierung nicht vorgegeben habe, man denke z.B. and den Kreis bzw. die Sphäre \bgroup\color{demo}$ X:=\{x:\Vert x\Vert=1\}$\egroup. Wir wollen trotzdem versuchen die lokalen Extrema von \bgroup\color{demo}$ f:X\to \mathbb{R}$\egroup zu lokalisieren und zwar ohne eine Parametrisierung von \bgroup\color{demo}$ X$\egroup benutzen zu müssen. Die Idee dabei ist bereits für eine Kurve \bgroup\color{demo}$ X\subseteq \mathbb{R}^2$\egroup zu erkennen. Ein Punkt \bgroup\color{demo}$ x_0\in X$\egroup ist höchstens dann extremal für eine Funktion \bgroup\color{demo}$ f$\egroup, wenn die Kurve die entsprechende Höhenschichtlinie \bgroup\color{demo}$ \{x:f(x)=f(x_0)\}$\egroup im Punkt \bgroup\color{demo}$ x_0$\egroup berührt. Die Tangente an die Höhenschichtlinie steht normal auf den Gradienten \bgroup\color{demo}$ \operatorname{grad}f(x_0)$\egroup. Wenn also \bgroup\color{demo}$ X$\egroup durch eine Gleichung \bgroup\color{demo}$ X:=\{x:g(x)=0\}$\egroup gegeben ist, dann muß ebenfalls die Tangente an \bgroup\color{demo}$ X$\egroup in \bgroup\color{demo}$ x_0$\egroup normal auf den Gradienten \bgroup\color{demo}$ \operatorname{grad}g(x_0)$\egroup stehen, und somit \bgroup\color{demo}$ \operatorname{grad}f(x)^\perp=\operatorname{grad}g(x)^\perp$\egroup gelten, d.h. \bgroup\color{demo}$ \operatorname{grad}f(x_0)$\egroup proportional zu \bgroup\color{demo}$ \operatorname{grad}g(x_0)$\egroup sein, d.h. ein \bgroup\color{demo}$ \lambda \in\mathbb{R}$\egroup existieren mit \bgroup\color{demo}$ f'(x_0)=\lambda \cdot g'(x_0)$\egroup. Einen entsprechenden allgemeineren Satz für \bgroup\color{demo}$ X$\egroup die durch mehrere Gleichungen (also eine Vektor-wertige Gleichung \bgroup\color{demo}$ X=\{x:g(x)=0\}$\egroup mit \bgroup\color{demo}$ g:E\to F$\egroup) beschrieben werden ist der folgende:

animation animation




19.3.5 Proposition über Lagrange Multiplikatoren.
Es sei \bgroup\color{demo}$ f:E\supseteq U\to\mathbb{R}$\egroup und \bgroup\color{demo}$ g:E\supseteq U\to F$\egroup, beide \bgroup\color{demo}$ C^1$\egroup. Falls \bgroup\color{demo}$ x\in U$\egroup unter der Nebenbedingung \bgroup\color{demo}$ g(x)=0$\egroup ein lokales Extremum von \bgroup\color{demo}$ f$\egroup ist und falls \bgroup\color{demo}$ g'(x)$\egroup surjektiv ist, so ist \bgroup\color{demo}$ f'(x)=\lambda \o g'(x)$\egroup für ein \bgroup\color{demo}$ \lambda \in L(F,\mathbb{R})$\egroup (dieses heißt Lagrange-Multiplikator).

Beweis. O.B.d.A. sei \bgroup\color{demo}$ x=0$\egroup. Es sei \bgroup\color{demo}$ E_1$\egroup der Kern von \bgroup\color{demo}$ g'(0)$\egroup und \bgroup\color{demo}$ E_2$\egroup ein Komplementärraum zu \bgroup\color{demo}$ E_1$\egroup also z.B. das orthogonale Komplement \bgroup\color{demo}$ E_2:=E_1^\perp$\egroup. Dann ist \bgroup\color{demo}$ E\cong E_1\oplus E_2$\egroup und wir bezeichnen mit \bgroup\color{demo}$ \tilde g:E_1\times E_2\to F$\egroup die Abbildung \bgroup\color{demo}$ \tilde g(x_1,x_2):= g(x_1+x_2)$\egroup und analog \bgroup\color{demo}$ \tilde f(x_1,x_2):=f(x_1+x_2)$\egroup. Dann ist \bgroup\color{demo}$ \d _j\tilde g(0)=g'(0)\vert _{E_j}$\egroup und insbesonders ist \bgroup\color{demo}$ \d _2 g(0)$\egroup injektiv, da \bgroup\color{demo}$ g'(0)$\egroup nur auf \bgroup\color{demo}$ E_1$\egroup verschwindet, und ebenso surjektiv, denn aus \bgroup\color{demo}$ w=g'(0)(v)$\egroup folgt für \bgroup\color{demo}$ v_1+v_2:=v$\egroup die Gleichung \bgroup\color{demo}$ w=g'(0)(v_1+v_2) =0+g'(0)(v_2)=\d _2 \tilde g(0)(v_2)$\egroup (Man sieht auch leicht, daß \bgroup\color{demo}$ \d _1\tilde g(0)=0$\egroup). Aus dem Satz über implizite Abbildungen folgt die Existenz einer lokal eindeutigen \bgroup\color{demo}$ C^1$\egroup-Lösung \bgroup\color{demo}$ h:E_1 \to E_2$\egroup der impliziten Gleichung \bgroup\color{demo}$ \tilde g(x_1,h(x_1))=0$\egroup, d.h. \bgroup\color{demo}$ g^{-1}(0)$\egroup läßt sich lokal durch den Graph einer Abbildung \bgroup\color{demo}$ h:E_1 \to E_2$\egroup parametrisieren läßt. Damit ist das Extremalproblem für \bgroup\color{demo}$ f$\egroup mit Nebenbedingung \bgroup\color{demo}$ g$\egroup auf das gewöhnliche Extremalproblem für \bgroup\color{demo}$ x_1\mapsto \tilde f(x_1,h(x_1))$\egroup ohne Nebenbedingung zurückgeführt. Da \bgroup\color{demo}$ \d _2\tilde g(0)$\egroup invertierbar ist können wir \bgroup\color{demo}$ \lambda :=\d _2\tilde f(0)\o\d _2\tilde g(0)^{-1}:F\to E_2\to \mathbb{R}$\egroup setzen, und erhalten \bgroup\color{demo}$ \d _2 \tilde f(0) = \lambda \o\d _2 \tilde g(0)$\egroup Nach (19.3.3) muß die Ableitung \bgroup\color{demo}$ \d _1 \tilde f(0)+\d _2\tilde f(0)\o h'(0)$\egroup von \bgroup\color{demo}$ x_1\mapsto \tilde f(x_1,h(x_1))$\egroup bei 0 verschwinden. Ebenso folgt durch Differenzieren von \bgroup\color{demo}$ \tilde g(x_1,h(x_1))=0$\egroup, daß die Ableitung \bgroup\color{demo}$ \d _1 \tilde g(0)+\d _2\tilde g(0)\o h'(0)$\egroup verschwindet, und da \bgroup\color{demo}$ \d _2\tilde g(0)$\egroup invertierbar ist, ist \bgroup\color{demo}$ h'(0)=-\d _2\tilde g(0)^{-1}\o\d _1\tilde g(0)$\egroup (Wegen \bgroup\color{demo}$ \d _1\tilde g(0)=0$\egroup ist \bgroup\color{demo}$ h'(0)=0$\egroup). Somit ist \bgroup\color{demo}$ \d _1 \tilde f(0)= -\d _2\tilde f(0)\o h'(0)
=\d _2\tilde f(0)\o\d _2\tilde g(0)^{-1}\o\d _1\tilde g(0)=\lambda \o\d _1\tilde g(0)$\egroup (Und dies ist 0). Also ist insgesamt \bgroup\color{demo}$ \tilde f'(0) =\lambda \o\tilde g'(0)$\egroup, und somit auch \bgroup\color{demo}$ f'(0)=\lambda \o g'(0)$\egroup, da \bgroup\color{demo}$ (x_1,x_2)\mapsto x_1+x_2$\egroup ein linearer Isomorphismus \bgroup\color{demo}$ E_1\times E_2\to E$\egroup ist der \bgroup\color{demo}$ f$\egroup und \bgroup\color{demo}$ g$\egroup in \bgroup\color{demo}$ \tilde f$\egroup und \bgroup\color{demo}$ \tilde g$\egroup übersetzt.     []

Image ..//pic-5-08.gif

Wenn \bgroup\color{demo}$ F=\mathbb{R}^n$\egroup ist, dann ist \bgroup\color{demo}$ \lambda $\egroup durch \bgroup\color{demo}$ (\lambda _i)_i\in \mathbb{R}^n$\egroup vermöge \bgroup\color{demo}$ \lambda (y_1,\dots,y_n):=\sum_{i=1}^n\lambda _i y_i$\egroup gegeben und somit muß ein relatives lokales Extremum \bgroup\color{demo}$ x$\egroup folgendes erfüllen

\bgroup\color{demo}$\displaystyle f'(x) = \sum_{i=1}^n \lambda _i g_i'(x)$\egroup    und \bgroup\color{demo}$\displaystyle g(x) = 0,
$\egroup

wobei \bgroup\color{demo}$ g(x)=:(g_1(x),\dots,g_n(x))$\egroup ist. Ist zusätzlich \bgroup\color{demo}$ E=\mathbb{R}^m$\egroup so bedeutet dies für ein lokales Extremum \bgroup\color{demo}$ x=(x_1,\dots,x_m)$\egroup folgendes (im allgemeinen nur bzgl. der \bgroup\color{demo}$ \lambda _i$\egroup lineare) Gleichungssystem von \bgroup\color{demo}$ n+m$\egroup Gleichungen in den \bgroup\color{demo}$ n+m$\egroup vielen Variablen \bgroup\color{demo}$ (x_1,\dots,x_m;\lambda _1,\dots,\lambda _n)$\egroup:

$\displaystyle \d _j f(x)$ $\displaystyle = \sum_{i=1}^n \lambda _i  \d _j g_i(x)$   für $\displaystyle j=1,\dots,m$    
$\displaystyle g_i(x)$ $\displaystyle = 0$   für $\displaystyle i=1,\dots,n$    

Da der letzte Satz nur eine notwendige Bedingung liefert, benötigt man zusätzliche Argumente um den Nachweis eines lokalen Extremums zu erbringen. Das kann z.B. sein, daß die Nebenbedingung eine kompakte Menge beschreibt und die stetige Funktion \bgroup\color{demo}$ f$\egroup somit ein Maximum und ein Minimum besitzen muß.


19.3.7 Beispiele für Extremalwerte unter Nebenbedingung.

  1. Gesucht sind die Extremalwerte von $ f(x,y,z):=5x+y-3z$ auf dem Durchschnitt der Ebene $ x+y+z=0$ mit der Sphäre $ x^2+y^2+z^2=1$. Die Abbildung $ g$ ist in diesem Beispiel $ g=(g_1,g_2):\mathbb{R}^3\to\mathbb{R}^2$ mit $ g_1(x,y,z):=x+y+z$ und $ g_2(x,y,z):=x^2+y^2+z^2-1$. Es ist somit folgendes Gleichungssystem in $ (x,y,z;\lambda _1,\lambda _2)$ zu lösen:

    $\displaystyle 5=\d _1 f(x,y,z)$ $\displaystyle = \lambda _1 \d _1 g(x,y,z) + \lambda _2 \d _1 g(x,y,z) = \lambda _1 1+\lambda _2 2x$    
    $\displaystyle 1=\d _2 f(x,y,z)$ $\displaystyle = \lambda _1 \d _2 g(x,y,z) + \lambda _2 \d _2 g(x,y,z) = \lambda _1 1+\lambda _2 2y$    
    $\displaystyle -3=\d _3 f(x,y,z)$ $\displaystyle = \lambda _1 \d _3 g(x,y,z) + \lambda _2 \d _3 g(x,y,z) = \lambda _1 1+\lambda _2 2z$    
    0 $\displaystyle = g_1(x,y,z) = x+y+z$    
    0 $\displaystyle = g_2(x,y,z) = x^2+y^2+z^2-1$    

    Die einzigen zwei Lösungen sind $ (\pm \frac{1}{\sqrt{2}},0,\mp\frac{1}{\sqrt{2}};\pm 2\sqrt{2},1)$. Einsetzen in $ f$ zeigt, daß $ (\frac{1}{\sqrt{2}},0,-\frac{1}{\sqrt{2}})$ die Stelle eines lokalen (und somit auch globalen) Maximums und $ (-\frac{1}{\sqrt{2}},0,\frac{1}{\sqrt{2}})$ die eines globalen Minimums ist. Genauer, da die Sphäre und somit auch der Schnitt mit der Ebene kompakt ist und $ f$ stetig ist, muß nach (16.3.6) ein Maximum und ein Minimum von $ f$ unter dieser Nebenbedingung existieren. Nach (19.3.5) sind die einzigen möglichen Punkte dafür aber die gerade berechneten, und da $ f$ auf dem ersten Punkt einen größeren Wert als am zweiten hat, muß der erste die Stelle des Maximums und der zweite jene des Miniums sein.

    Image ..//pic-5-06.gif
  2. Sei $ A\in L(E,E)$ gegeben. Gesucht seien die Extremalwerte von $ \Vert Ax\Vert$ (oder äquivalent von $ \Vert Ax\Vert^2=\langle Ax,Ax\rangle$) unter der Nebenbedingung $ \Vert x\Vert=1$ (äquivalent von $ \Vert x\Vert^2=\langle x,x\rangle$). Die Methode der Lagrangemultiplikatoren liefert:

    $\displaystyle 2\langle A^tAx,v\rangle=\langle Ax,Av\rangle+\langle Av,Ax\rangle$ $\displaystyle = \lambda (\langle x,v\rangle+\langle v,x\rangle)=2 \langle \lambda  x,v\rangle\quad\forall v$    
    $\displaystyle \langle x,x\rangle$ $\displaystyle = 1$    

    Somit ist $ A^tAx=\lambda  x$ und $ \Vert x\Vert=1$ und folglich $ x$ ein normierter Eigenvektor von $ A^tA$ zum Eigenwert $ \lambda $. Ist $ \dim(E)=2$, dann liefert die beiden Eigenwerte $ \lambda $ das Maximum und das Minimum von $ \Vert Ax\Vert^2$ und die Bilder $ A(v)$ der zugehörigen normierten Eigenvektoren $ v$ sind gerade die Halbachsen der durch $ \{A(x):\Vert x\Vert=1\}$ beschrieben Ellipse. Die Längen der Halbachsen sind $ \Vert A(x)\Vert=\sqrt{\langle A(x),A(x)\rangle}=
\sqrt{\lambda \langle x,x\rangle}=\sqrt{\lambda }\Vert x\Vert=\sqrt{\lambda }$.

    Andererseits können wir auch die Quadrik $ X:=\{x:\Vert Ax\Vert=1\}$ untersuchen. Die Punkte $ x\in X$ mit extremalen Abstand $ \Vert x\Vert$ zu 0 sind wie folgt zu erhalten. Wir setzen $ v:=\frac1{\Vert x\Vert}x$ und erhalten $ \Vert v\Vert=1$ und $ \Vert A(v)\Vert=\frac{\Vert Ax\Vert}{\Vert x\Vert}=\frac1{\Vert x\Vert}$. Wir suchen also unter allen $ v$ mit $ \Vert v\Vert=1$ jene, für welche $ \Vert A(v)\Vert$ extremal wird, dies sind nach obigen genau die normierten Eigenvektoren $ v$ von $ A^tA$ und die zugehörigen Eigenwerte sind $ \lambda =\frac1{\vert x\vert}$. Die Punkte $ x\in X$ mit extremalen Abstand zu 0 sind also gerade $ \frac1{\lambda }v$ mit Norm $ \frac1{\lambda }$. Die Eigenvektoren $ v$ beschreiben also die Richtungen der Hauptachsen der Quadrik (d.h. im $ \mathbb{R}^2$ der Ellipse oder Hyperbel) $ X$.

    Image ..//pic-5-07.gif

Andreas Kriegl 2002-07-01